You are on page 1of 66

TEST - 15

IA
S

1 Which of the following correctly point out the geo-strategic significance of the Chumbi
valley?
1. It is at the intersection of Sikkim, Bhutan and Tibet in the Himalayas.
2. Two main passes including the Nathu la pass between India and China open up here.
Which of the above is/are correct?
A. 1 only
B. 2 only
C. Both 1 and 2
D. None
User Answer :
Correct Answer : C
Answer Justification :

TS

Justification:Statement 1: It is in Tibet at the intersection of India (Sikkim), Bhutan


and China (which claims Tibet) in the Himalayas. So, it is crucial to settling the
territorial disputes between India and China in Tibet.

Statement 2: The Nathu La Pass and Jelep La Pass open up here. Nathu La is one of
the two open trading border posts between China and India which was closed
following the 1962 war and re-opened in 2006.

Q Source:Physical features of India: Chapter 1: India Yearbook 2016

IN

SI

2 Consider the following statements.


1. Assertion (A): A state legislature cannot impose any taxes on the sale or purchase of
goods on its own.
2. Reason (R): A state legislature needs the approval of the President for imposing any
tax.
In the context of the above, which of these is correct?
A. A is correct, and R is an appropriate explanation of A.
B. A is correct, but R is not an appropriate explanation of A.
C. A is incorrect, but R is correct.
D. Both A and R are incorrect.
User Answer :
Correct Answer : D
Answer Justification :
Justification: A state legislature can impose taxes on professions, trades, callings
and employments, sale or purchase of goods (other than newspapers) etc.
However, a tax imposed on the sale or purchase of goods declared by Parliament to

(C) Insights Active Learning. | All rights reserved.

www.insightsias.com

TEST - 15

be of special importance in inter-state trade and commerce is subject to the


restrictions and conditions specified by the Parliament.
Presidential assent is not generally required for introduction of taxes by a state

IA
S

Q Source: Chapter 14: Indian Polity: M Laxmikanth

TS

3 They are a series of parallel ridges arcing through Assam, Nagaland, Mizoram, Burma
and are submerged in the Bay of Bengal for a long stretch emerging again in the form of
the Andaman and Nicobar Islands. The mountains are
A. Namcha Barwa Ranges
B. Trans-Alay Range
C. Dhaulagiri Mountains
D. Arakan Mountains
User Answer :
Correct Answer : D
Answer Justification :

Learning: The Arakan Mountains (also called Rakhine ranges) and the parallel arcs
to the west and east were formed by compression as the Indian Plate collided with
the Eurasian Plate approximately along the boundary between India and Nepal.

SI

The Arakan Mountains divide the Rakhine coast from the rest of Burma, and thus
have acted as a barrier between the peoples of central Burma and those of the Indian
subcontinent.

IN

The Arakan Mountains act as a barrier to the south-western monsoon rains and thus
shield the central Myanmar area, making their western slopes extraordinarily wet
during the monsoon.
Q Source: Physical features of India: Chapter 1: India Yearbook 2016

4 Consider the following about Madan Mohan Malaviya.


1. He took a vow not to join any legislative council.
2. He was the founder of Ganga Mahasabha.
3. He opposed the separate electorates for Muslims under the Lucknow Pact of 1916.
4. He was a President of the Indian National Congress (INC).
Select the correct answer using the codes below.
A. 2, 3 and 4 only
B. 4 only

(C) Insights Active Learning. | All rights reserved.

www.insightsias.com

TEST - 15

C. 1 and 3 only
D. 1, 2, 3 and 4
User Answer :
Correct Answer : A
Answer Justification :

IA
S

Justification: Statement 1: He was a member of the Imperial Legislative Council


from 1912 and when in 1919 it was converted to the Central Legislative Assembly
he remained its member till 1926.
Statement 2: He founded it at Haridwar in 1905. He was a member of the Hindu
Mahasabha.

TS

Statement 3: He was a moderate leader and opposed communal politics by the


British. He was an important figure in the Non-cooperation movement. However, he
was opposed to the politics of appeasement by Congress and its participation in the
Khilafat movement.

Statement 4: Malaviya was the President of the Indian National Congress on two
occasions (1909, 1918). He left Congress in 1934.

Q Source: Next to Cover page: India Yearbook 2016

IN

SI

5 Consider the following statements.


1. Assertion (A): No river originating in Rajasthan meets the sea.
2. Reason (R): A large part of Rajasthan is desert.
In the context of the above, which of these is correct?
A. A is correct, and R is an appropriate explanation of A
B. A is correct, but R is not an appropriate explanation of A.
C. A is incorrect, but R is correct.
D. Both A and R are correct independently.
User Answer :
Correct Answer : C
Answer Justification :
Justification: Sabarmati River is one example that originates in Rajasthan
(Udaipur) and meets the Arabian Sea.
Only a few rivers in Rajasthan do not drain into the sea. They drain into salt lakes
and get lost in sand with no outlet to sea.

(C) Insights Active Learning. | All rights reserved.

www.insightsias.com

TEST - 15

Besides these, there are the Desert Rivers which flow for some distance and are lost
in the desert. These are Luni, Machhu, Rupen, Saraswati, Banas, Ghaggar and
others.
Q Source:Physical features of India: Chapter 1: India Yearbook 2016

TS

IA
S

6 Which of the following have been introduced by foreigners in India?


1. Potatoes
2. Tomato
3. Red Chilli
4. Tea and Coffee
5. Peanuts and Almond
Select the correct answer using the codes below.
A. 3, 4 and 5 only
B. 4 and 5 only
C. 1, 2, 3 and 4 only
D. 1, 2, 3, 4 and 5
User Answer :
Correct Answer : C
Answer Justification :

Learning: Statement 1: When the Dutch came to India, they introduced the culture
of potatoes, and from them the British received various kinds of potatoes.

SI

Statement 2: Originating in South America, it may have been introduced as late as


1850 into India.
Statement 3: It arrived in India during 16th century replacing Pippali (long pepper).

IN

Statement 4: Tea was first introduced into India by the British, in an attempt to
break the Chinese monopoly on tea.
The first record of coffee growing in India is following the introduction of coffee
beans from Yemen by Baba Budan to the hills of Chikmagalur in 1670.
Q Source: Revision Previous Tests Syllabus: 11th Geography NCERT: India,
People and Economy

7 Consider the following about the protocol followed while hoisting the National Flag.
1. The Flag should never touch the ground or water.

(C) Insights Active Learning. | All rights reserved.

www.insightsias.com

TEST - 15

Indian citizens can fly the flag even at night.


The Flag should not hold any objects other than flower petals before unfurling.
No lettering should be inscribed on the flag.
The flag cannot be used in uniforms.
Select the correct answer using the codes below.
A. 2 and 4 only
B. 3, 4 and 5 only
C. 1, 2 and 3 only
D. 1, 2, 3 and 4 only
User Answer :
Correct Answer : D
Answer Justification :

IA
S

2.
3.
4.
5.

TS

Learning: The original flag code also forbade use of the flag on uniforms, costumes
and other clothing.
In 2005, the Government of India amended the code to allow some forms of usage.
The amended code forbids usage in clothing below the waist and on undergarments,
and forbids embroidering onto pillowcases, handkerchiefs or other dress material.

Display and usage of the flag is governed by the Flag Code of India, 2002
(successor to the Flag Code - India, the original flag code); the Emblems and Names
(Prevention of Improper Use) Act, 1950; and the Prevention of Insults to National
Honour Act, 1971.

SI

Q Source: National Flag: Chapter 2: India Yearbook 2016

IN

8 World Consumer Rights Day is observed on 15th March to commemorate the historic
address given by the then US President John F Kennedy to the US congress in 1962. In
India, 24th December is observed as National Consumer Day. On this day
A. Consumer Protection Act, 1986 had received the assent of the President.
B. Dandi March started which was the first ever consumer rights movement in
India.
C. First consumer dispute was resolved by the Supreme Court.
D. The first ever consumer case came to light in independent India.
User Answer :
Correct Answer : A
Answer Justification :
Learning: 2016 Theme of the World Consumer Rights Day: "Antibiotics off the
menu" - The theme is a campaign to stop the sale of meat raised with the use of

(C) Insights Active Learning. | All rights reserved.

www.insightsias.com

TEST - 15

antibiotics important to human medicine.


It seeks to spread awareness about antibiotic resistance driven by overuse of
antibiotics in agriculture to promote faster growth and to prevent diseases rather
than treat diseases.

IA
S

Q Source: http://pib.nic.in/newsite/PrintRelease.aspx?relid=133634

TS

9 In the Indian National Saka Calendar, the months in the first half of the year all have 31
days to take into account
A. Change of seasons
B. Slower movement of the sun across the ecliptic
C. Old traditions and festivals
D. Orbital velocity of the Moon
User Answer :
Correct Answer : B
Answer Justification :

Justification: The ecliptic is the apparent path of the Sun on the celestial sphere. It
is coplanar (same plane) with the orbit of Earth around the Sun.

The Sun seems to move against the background stars as seen from the orbiting
Earth. The ecliptic is the path the Sun appears to trace through the stars.

SI

Because Earth takes one year to orbit the Sun, the apparent position of the Sun also
takes the same length of time to make a complete circuit of the ecliptic.

IN

But, the actual speed with which Earth orbits the Sun varies slightly during the year,
so the speed with which the Sun seems to move along the ecliptic also varies. This is
taken into account by the Saka Calendar and the first six months are adjusted for the
apparent slow movement of the Sun in the ecliptic.
Q Source: National Calendar: Chapter 2: India Yearbook 2016

10 The provision that Speaker does not vacate her office until immediately before the first
meeting of the House after dissolution is found in the
A. Parliamentary rules and procedure
B. Constitution of India
C. Representation of People of India Act, 1950
D. Conventions of the Indian Parliament

(C) Insights Active Learning. | All rights reserved.

www.insightsias.com

TEST - 15

User Answer :
Correct Answer : B
Answer Justification :
Justification: Article 94 of the constitution deals with the vacation/ resignation/
removal from the offices of the Speaker and Deputy Speaker.

IA
S

It provides that whenever the House of the People is dissolved, the Speaker shall not
vacate his office until immediately before the first meeting of the House of the
People after the dissolution.
Q Source: Table 3.1: Chapter 3: India Yearbook 2016

IN

SI

TS

11 Which of the following departments do NOT belong to the Union Ministry of Home
Affairs?
1. Department of Administrative Reforms
2. Department of Defence Production
3. Department of States
4. Department of Home
5. Department of Ex-servicemen Welfare
6. Department of Official Language
Select the correct answer using the codes below.
A. 1, 2, 4 and 5 only
B. 1, 2 and 5 only
C. 2, 3, 4 and 5 only
D. 1 and 6 only
User Answer :
Correct Answer : B
Answer Justification :
Justification: 2 and 5 are in Ministry of Defence. 1 is in the Ministry of Personnel,
Public Grievances and Pensions.
Departments under Ministry of Home Affairs are:
Department of Internal Security (Aantarik Suraksha Vibhag)
Department of States (Rajya Vibhag)
Department of Official Language (Raj Bhasha Vibhag)
Department of Home (Grih Vibhag)
Department of Jammu and Kashmir Affairs (Jammu tatha Kashmir Vibhag)
Department of Border Management (Seema Prabandhan Vibhag)

(C) Insights Active Learning. | All rights reserved.

www.insightsias.com

TEST - 15

Q Source: Chapter 3: India Yearbook 2016

TS

IA
S

12 As per the Aadhaar (Target Delivery of Financial and Other Subsidies, Benefits and
Services) Bill, 2016 passed by Lok Sabha
1. Aadhaar can now be used as a proof of citizenship or domicile in India.
2. Biometric information collected during the Aadhaar enrolment can now be freely
used by the Police for investigating crimes.
Which of the above is/are correct?
A. 1 only
B. 2 only
C. Both 1 and 2
D. None
User Answer :
Correct Answer : D
Answer Justification :

Justification: Statement 1: Use of Aadhaar number will be for: (i) Verifying the
identity of a person receiving a subsidy or a service. (ii) as a proof of identity of the
Aadhaar number holder for any purpose asked by any public or private entity. (iii) It
cannot be a proof of citizenship or domicile.

SI

Statement 2: Biometric information will be used only for Aadhaar enrolment and
authentication and for no other purpose. Such information will not be shared with
anyone, nor displayed publicly, except for purposes specified by regulations, i.e.
national security and court orders.

IN

Q Source:
http://www.thehindu.com/news/national/lok-sabha-passes-aadhaar-bill/article83419
08.ece

13 The Krishonnati Yojana is a new umbrella scheme submerging which of the following
existing scheme(s)?
1. Soil Health Card scheme
2. Paramparagat Krishi Vikas Yojana
3. National Food Security Mission
Select the correct answer using the codes below.
A. 1 and 2 only
B. 3 only
C. 2 and 3 only
D. 1, 2 and 3

(C) Insights Active Learning. | All rights reserved.

www.insightsias.com

TEST - 15

User Answer :
Correct Answer : B
Answer Justification :
Justification: National Food Security Mission was launched in 2007-08 to increase
the production of rice, wheat and pulses

IA
S

Paramparagat Krishi Vikas Yojana is a cluster based programme to encourage the


farmers for promoting organic farming.
Under the Soil Health Card Scheme, Soil Health Card (SHC) will be provided to all
farmers in the country at an interval of three years so as to enable them to apply
appropriate recommended dosages of nutrients for crop production and improving
soil health and its fertility.

TS

Q Source: Chapter 4: India Yearbook 2016

IN

SI

14 Which of the following statements is INCORRECT about Indicative planning?


1. Mixed economies can follow indicative planning.
2. The state does not play any role in indicative planning.
3. It signals liberalization of market forces.
4. No state owned companies are allowed to operate in the economy.
Select the correct answer using the codes below.
A. 2, 3 and 4 only
B. 1 and 2 only
C. 2 and 4 only
D. 4 only
User Answer :
Correct Answer : C
Answer Justification :
Justification: Statement 1 and 2: Capitalist economies do not involve the state in
the planning process. And, in communist economies the planning is imperative not
indicative in nature. So, mixed economies are fit for indicative planning where a
plan making body (from the State) sets broad numerical targets to be achieved with
the help of market forces.
Statement 3: Unlike a centrally planned economy indicative planning works through
the market (price system) rather than replaces it. So, it is a move towards
liberalization of market forces

(C) Insights Active Learning. | All rights reserved.

www.insightsias.com

TEST - 15

Statement 4: India followed indicative planning, but the PSUs still continue to
operate.
Q Source: Types of Planning: Chapter 4: Indian Economy - Ramesh Singh

TS

IA
S

15 Which of the following is/are member countries of BIMSTEC (Bay of Bengal Initiative
on Multi Sectoral Technical and Economic Cooperation)?
1. Nepal
2. Bhutan
3. Myanmar
4. Thailand
5. Sri Lanka
6. China
Select the correct answer using the codes below
A. All except 3 and 4
B. All except 1 and 2
C. All except 5 and 6
D. All except 6
User Answer :
Correct Answer : D
Answer Justification :

Learning: It is the sub-regional group of seven countries in South Asia and South
East Asia.

SI

They are India, Nepal, Bangladesh, Bhutan, Sri Lanka (from South Asia) and
Myanmar, Thailand (from South East Asia).

IN

It was established in 1997. Headquarters is Dhaka, Bangladesh.


It also covers cooperation in commerce, technology, investment, agriculture,
tourism, human resource development, fisheries, transport and communication,
textiles, leather etc.
Q Source: Revision Previous Tests Syllabus: International and Regional
organizations

16 "The Economic History of India" written in the 20th century which examined in minute
detail the entire economic record of colonial rule since 1757 was written by
A. Mahadev Govind Ranade

(C) Insights Active Learning. | All rights reserved.

www.insightsias.com

10

TEST - 15

IA
S

B. G. Subramaniya lyer
C. G.K. Gokhale
D. Romesh Chandra Dutt
User Answer :
Correct Answer : D
Answer Justification :
Learning: The book explores the drain of wealth from India. It shows how specific
policies of British in the sphere of railways, cotton, plantation agriculture,
handicrafts led to the ruin of Indians and India's economy.
Romesh Chandra Dutt was active in moderate nationalist politics and was an active
Congressman in that party's initial phase.

TS

He was also a president of the Indian National Congress in 1899.


Q Source: Page 68: Chapter 7: Bipin Chandra: India's struggle for Independence

IN

SI

17 Consider the following about 'One MP - One Idea Scheme'.


1. It was initiated by the Ministry of Parliamentary affairs.
2. The District administration submits innovative ideas to MPs who then forward it to
the Union Cabinet for ratification and implementation.
3. Ideas must be funded by the concerned MPs through MPLADS funds.
Select the correct answer using the codes below.
A. 1 and 2 only
B. 2 only
C. 2 and 3 only
D. None of the above
User Answer :
Correct Answer : D
Answer Justification :
Justification: Statement 1: It was announced by the Ministry of Statistics and
Programme Implementation under the Member of Parliament Local Area
Development Scheme (MPLADS).
Statement 2: Based on the innovative ideas received from the local people regarding
developmental projects, a 'One MP - One Idea' Competition can be held in each Lok
Sabha constituency annually to select the three best innovations for cash awards and
certificate of appreciation for next five best innovations.

(C) Insights Active Learning. | All rights reserved.

www.insightsias.com

11

TEST - 15

A committee headed by DM evaluates these ideas.


Statement 3: MPs award Certificate of Honour along with cash award of Rs. 2.5
lakhs, Rs. 1.5 lakhs and Rs. 1 lakh to the 1st , 2nd and 3rd best innovative solutions
respectively through their MPLAD funds.

IA
S

Lead banks and financial institutions fund local innovations.


An overview presentation on the 'One MP - One Idea' initiative can be viewed here
http://innovationcouncilarchive.nic.in/index.php?option=com_content&view=article
&id=343&Itemid=121

TS

Q Source: MPLADS: Chapter 5: Indian Economy - Ramesh Singh

SI

18 Consider the following about Tattwabodini Sabha.


1. It was a splinter group of the Brahmo Samaj.
2. It was founded by Debendranath Tagore.
3. Its main objective was to promote a rational and humanist form of Hinduism based
on the Vedanta.
Select the correct answer using the codes below.
A. 1 and 2 only
B. 3 only
C. 2 and 3 only
D. 1, 2 and 3
User Answer :
Correct Answer : D
Answer Justification :

IN

Learning: As per some commentators, "Their view, at least in the early years, was
that the world is created by God, and all things within it are pathways to knowledge
of Brahman, the Ultimate Self, and the ultimate goal. Similarly, they saw that
material wealth, if made and possessed with the correct intention - that of helping
society and others - was in fact not only ethically sound, but an utter necessity for
harmonious society."
In 1859, the Sabha was dissolved back into the Brahmo Samaj by Debendranath
Tagore.
Q Source: Page 65: Chapter 6: Bipin Chandra: India's struggle for Independence

(C) Insights Active Learning. | All rights reserved.

www.insightsias.com

12

TEST - 15

TS

IA
S

19 Which of the following measures may help tackle inflation?


1. Import of goods which are in short-supply
2. Technological innovations that reduce the production cost of goods
3. Improving Supply-side infrastructure
4. Tight monetary policy
5. Cracking down on illegal hoarding of sensitive items
Select the correct answer using the codes below.
A. 1, 3 and 4 only
B. 1, 2 and 5 only
C. 3, 4, 5 only
D. 1, 2, 3, 4 and 5
User Answer :
Correct Answer : D
Answer Justification :

Justification: Statement 1, 3 and 5: It has happened in India in the case of onion and
pulses and meeting the buffer stock norm of wheat. As a long-term measure,
governments go on to increase the production to matching the level of demand.
Storage, transportation, distribution, hoarding are the other aspects of price
management of this category.

Statement 2: For e.g. Cost of energy (electricity) has effectively come down due to
introduction of LED bulbs. Any possible inflation in energy prices are also taken
care by the low power consumption and electricity bills due to LEDs.

SI

Statement 4: It is basically intended to cut down the money supply in the economy
by siphoning out the extra money (as RBI increases the Cash Reserve Ratio of bank
in India)9 from the economy and by making money costlier (as RBI increases the
Bank Rate or Repo Rate in India).

IN

This is a short-term measure. In the long-run, the best way is to increase production
with the help of the best production practices.
Q Source: Chapter 7: Indian Economy - Ramesh Singh

20 Washington Consensus refers to


A. A set of economic reform prescriptions for developing countries
B. A strategy to tackle organized terrorism in the Middle-East
C. A set of prescriptions for ecologically sustainable mining in erstwhile
colonial African countries
D. A plan by UNFCCC to reverse climate change and save low lying island

(C) Insights Active Learning. | All rights reserved.

www.insightsias.com

13

TEST - 15

nations
User Answer :
Correct Answer : A
Answer Justification :

IA
S

Learning: It is considered to constitute the "standard" reform package promoted for


crisis-wracked developing countries by Washington, D.C.-based institutions such as
the International Monetary Fund (IMF), World Bank, and the US Treasury
Department.
It basically refers to a more general orientation towards a strongly market-based
approach such as macroeconomic stabilization, economic opening with respect to
both trade and investment, and the expansion of market forces within the domestic
economy.

TS

Q Source: Chapter 6: Indian Economy - Ramesh Singh

SI

21 Consider the following statements.


1. He started the first Tamil newspaper Swadeshmitram in 1882.
2. He founded 'The Hindu' newspaper and was also its editor till 1898.
The above refer to?
A. G. Subrahmaniya Iyer
B. Vaidyanatha Iyer
C. Alluri Sita Rama Raju
D. Chinnaswami Subramanya Bharathiyar
User Answer :
Correct Answer : A
Answer Justification :

IN

Learning: He was proprietor, editor and managing director of The Hindu


Subramania Iyer actively participated in the Indian Independence movement.
Subramania Iyer campaigned vehemently for reforms in Hindu society. He
supported widow remarriage and desired to abolish untouchability and child
marriages.
In 1898, Subramania Iyer relinquished his claims over 'The Hindu' and concentrated
his energies on Swadesamitran, the Tamil language newspaper which he had started
in 1882.

(C) Insights Active Learning. | All rights reserved.

www.insightsias.com

14

TEST - 15

Q Source: Page 79: Chapter 8: Bipin Chandra: India's struggle for Independence

TS

IA
S

22 Consumer Price Index (CPI) was published by the Government across various
categories like Industrial workers. How were these various categories useful from an
economic perspective?
1. They formed the basis for Pay Commission's recommendations.
2. They were used to deflate GDP of certain sectors to obtain real GDP of that sector.
3. They were used to revise minimum wages for agricultural labourers.
4. They showed the price impact on the most vulnerable segments of India.
Select the correct answer using the codes below.
A. 1, 3 and 4 only
B. 2 and 3 only
C. 4 only
D. 1, 2, 3 and 4
User Answer :
Correct Answer : D
Answer Justification :

Justification: It is the Old CPI index. New Index is published across Rural, Urban
and Combined categories.

SI

Statement 1: The Consumer Price Index for the industrial workers (CPI-IW)
specifies the government employees (other than banks' and embassies' personnel).
The wages/salaries of the central government employees are revised on the basis of
the changes occurring in this index, the dearness allowance (DA) is announced twice
a year.
When the Pay Commissions recommend pay revisions, the base is the CPI (IW).

IN

Statement 2: The Consumer Price Index for the Urban Non-Manual Emplyees (CPIUNME) is basically used for determining dearness allowances (DAs) of employees
of some foreign companies operating in India (i.e. airlines, communications,
banking, insurance, embassies, and other financial services).
It is also used under the Income Tax Act to determine capital gains and by the CSO
(Central Statistical Organisation) for deflating selected service sector's contribution
to the GDP at factor cost and current prices to calculate the corresponding figure at
constant prices.
Statement 3 and 4: The Consumer Price Index for Agricultural Labourers (CPI-AL)
is used for revising minimum wages for agricultural labourers in different states.

(C) Insights Active Learning. | All rights reserved.

www.insightsias.com

15

TEST - 15

The governments at the centre and states remain vigilant regarding the changes in
this index as it shows the price impact on the most vulnerable segment of the
society, this segment spends almost 75 per cent of its total income on the purchase
of food articles.

IA
S

Q Source: Chapter 7: Indian Economy - Ramesh Singh

TS

23 The President is bound by the aid and advice tendered by the Council of Ministers. This
provision is
A. Mandated by Representation of People of India Act
B. Followed as a convention of Parliamentary form of Government
C. An executive precedent followed since Independence
D. Mandated by a Constitutional amendment
User Answer :
Correct Answer : D
Answer Justification :

Learning: Article 74 provides for a council of ministers headed by the Prime


Minister to aid and advise the President in the exercise of his functions. The advice
so tendered is binding on the President

Unamended constitution had some scope of Presidential discretion where he was not
explicitly bound by the aid and advice of the council of Ministers.

SI

Q Source: Chapter 12: Indian Polity: M Laxmikanth

IN

24 Consider the following about President's rule that is imposed under Article 356 of the
Constitution.
1. It cannot be imposed without the written recommendation of the Governor of the
concerned state.
2. Every proclamation of President's rule must be approved by both the houses of
Parliament within a stipulated time.
Which of the above is/are correct?
A. 1 only
B. 2 only
C. Both 1 and 2
D. None
User Answer :
Correct Answer : B
Answer Justification :

(C) Insights Active Learning. | All rights reserved.

www.insightsias.com

16

TEST - 15

Learning: Statement 1: Article 356 empowers the President to issue a proclamation,


if he is satisfied that a situation has arisen in which the government of a state cannot
be carried on in accordance with the provisions of the Constitution.
Notably, the president can act either on a report of the governor of the state or
otherwise too (ie, even without the governor's report).

IA
S

Statement 2: A proclamation imposing President's Rule must be approved by both


the Houses of Parliament within two months from the date of its issue. If approved
by both the Houses of Parliament, the President's Rule continues for six months
Q Source: Chapter 16: Indian Polity: M Laxmikanth

SI

TS

25 Some inflation is necessary for boosting investments in the short-run. This happens
because
1. Higher inflation indicates higher demand in the economy
2. Higher inflation lowers the cost of credit for entrepreneurs
Which of the above is/are correct?
A. 1 only
B. 2 only
C. Both 1 and 2
D. None
User Answer :
Correct Answer : C
Answer Justification :

IN

Justification: Statement 1: Higher inflation is because demand has outstripped


supply. When this happens, entrepreneurs are motivated to expand production to
fulfil the excess demand. This boosts investment in the economy.
Statement 2: Inflation redistributes wealth from creditors to debtors i.e. lenders
suffer and borrowers benefit out of inflation. The concept has also been covered in
an earlier test. So, entrepreneurs (who borrow money for investing) benefit from
inflation in repaying their loans.
Q Source: Chapter 7: Indian Economy - Ramesh Singh

26 Which of the following cases in the Supreme Court of India is related to resolving the
conflict between Fundamental Rights and Directive Principles?
1. Golaknath case

(C) Insights Active Learning. | All rights reserved.

www.insightsias.com

17

2. Champakam Dorairajan case


3. Minerva Mills case
4. Bhikan Singh case
Select the correct answer using the codes below.
A. 1 and 3 only
B. 2, 3 and 4 only
C. 1 and 4 only
D. 1, 2 and 3 only
User Answer :
Correct Answer : D
Answer Justification :

IA
S

TEST - 15

TS

Learning: In the Champakam Dorairajan case, the Supreme Court ruled that in case
of any conflict between the Fundamental Rights and the Directive Principles, the
former would prevail.
In Golaknath case (1967) the Supreme Court ruled that the Parliament cannot take
away or abridge any of the Fundamental Rights, which are 'sacrosanct' in nature.

In the Minerva Mills case (1980), the Supreme Court also held that 'the Indian
Constitution is founded on the bedrock of the balance between the Fundamental
Rights and the Directive Principles.

Q Source: Chapter 8: Indian Polity: M Laxmikanth

IN

SI

27 The Arab League is a regional organization of Arab countries in and around North
Africa, the Horn of Africa and Arabia. Member countries of the Arab League are
1. Sudan
2. Iraq
3. Iran
4. Jordan
5. Libya
6. Somalia
Select the correct answer using the codes below
A. 2, 4 and 5 only
B. 1, 4 and 6 only
C. 3, 5 and 6 only
D. 1, 2, 4, 5 and 6 only
User Answer :
Correct Answer : D

(C) Insights Active Learning. | All rights reserved.

www.insightsias.com

18

TEST - 15

Answer Justification :
Learning: The Arab League occupies an area spanning around 14 million km and
counts 22 members, and 4 observer states. The 22 members today include 3 of the
largest African countries (Sudan, Algeria and Libya) and the largest country in the
Middle East (Saudi Arabia).

IA
S

Syria was suspended following the 2011 uprising, but its seat was later given to
Syrian opposition.
It was founded in 1945. It aimed to be a regional organisation of Arab states with a
focus to developing the economy, resolving disputes and coordinating political aims.

TS

Q Source: Revision Previous Tests Syllabus: International and Regional


organizations

IN

SI

28 Amrita Bazaar Patrika played a major role in the evolution and growth of Indian
journalism and and nurturing the Indian freedom struggle. Consider the following about it.
1. It is the oldest Indian-owned English daily.
2. It was started by Sisir Ghosh and Moti Lal Ghosh.
3. It was discontinued before the Quit India Movement.
Select the correct answer using the codes below.
A. 1 and 2 only
B. 2 and 3 only
C. 1 and 3 only
D. 1, 2 and 3
User Answer :
Correct Answer : A
Answer Justification :
Learning: It was launched in Bengali and later continue in English.
It used to be a nationalist newspaper during the British rule, discontinued its
publication from 1986.
It was the first Indian-owned English daily to go into investigative
journalism.
The Patrika had many brushes with Lord Curzon, the Viceroy of India at the
time of the Partition of Bengal (1905).
It referred to him as 'Young and a little foppish, and without previous training
but invested with unlimited powers.'

(C) Insights Active Learning. | All rights reserved.

www.insightsias.com

19

TEST - 15

Because of such editorials, the Press Act of 1910 was passed and Motilal
Ghosh was also charged with sedition.
Q Source: Page 83: Chapter 8: Bipin Chandra: India's struggle for Independence

TS

IA
S

29 The Union Government has launched an online marketing platform Mahila E-HAAT to
facilitate women entrepreneurs to sell their products to buyers. It is a joint initiative of
A. Department of Electronics and Information Technology and All India
Women's Federation (AIWF)
B. Union Women and Child Development Ministry and Rashtriya Mahila Kosh
(RMK)
C. Commission on the Status of Women (CSW) and Union Women and Child
Development Ministry
D. Department of Electronics and Information Technology and Self-Employed
Women's Association (SEWA)
User Answer :
Correct Answer : B
Answer Justification :

Learning: It is an initiative mainly for women across the country and is part of
Union Government's flagship 'Digital India' and 'Stand Up India' initiatives.

Its objective is to strengthen the financial inclusion of women entrepreneurs by


providing continued support to their creativity

IN

SI

The initiative is unique as for the first time government will help women to
sell products online.
Women can sell their products like cloths, bags, jewelleries through this web
portal.
It is open to all Indian women citizens above 18 years of age and women Self
Help Groups (SHGs) desiring for marketing their legal products/services. The
entire business on the e-Haat platform can be easily handled through mobile
phone.

Q Source: http://pib.nic.in/newsite/PrintRelease.aspx?relid=137415
30 Which of the following statements about Ishwarchandra Vidyasagar is correct?
1. He was a key figure of the Bengal Renaissance.
2. He took the initiative in proposing and pushing the Widow Remarriage Act 1856.
Which of the above is/are correct?

(C) Insights Active Learning. | All rights reserved.

www.insightsias.com

20

TEST - 15

IA
S

A. 1 only
B. 2 only
C. Both 1 and 2
D. None
User Answer :
Correct Answer : C
Answer Justification :
Learning: He also demonstrated that the system of polygamy without restriction
was not sanctioned by the ancient Hindu Shastras.

TS

He reconstructed the Bengali alphabet and reformed Bengali typography into an


alphabet of twelve vowels and forty consonants. He contributed significantly to
Bengali and Sanskrit literature.Vidyasagar's "Barna Porichoy" is still considered a
classic.
Q Source: Page 58: Chapter 6: Bipin Chandra: India's struggle for Independence

IN

SI

31 Consider the following.


1. The economy heats up and a demand-supply lag is visible.
2. There can be shortage of investible capital and savings
3. A seller's market may be created.
The above may be associated with which of the following phases of business cycle?
A. Depression
B. Boom
C. Recession
D. Stagflation
User Answer :
Correct Answer : B
Answer Justification :
Justification: Statement 2: Due to a mismatch between supply and demand, and
higher consumption (and lower savings), there can be a shortage of investible
capital.
Due to this the economy heats up and supply-side bottlenecks become clearly
visible. So, statement 1 is also associated here.
Overheating of an economy occurs when its productive capacity is unable to keep
pace with growing aggregate demand leading to inflation.

(C) Insights Active Learning. | All rights reserved.

www.insightsias.com

21

TEST - 15

Statement 3: A seller's market is an economic situation in which goods or shares are


scarce and sellers can keep prices high. During boom as demand outstrips supply,
the situation can often be seen across.

IA
S

Q Source: Chapter 7 - Section B - Business Cycle: Indian Economy - Ramesh


Singh

TS

32 It has been said that the merits of the ASEAN Way might "be usefully applied to global
conflict management". What is called as the 'ASEAN Way'?
A. It reflects the spiritual life style of ASEAN members countries.
B. Involving third party countries in bilateral negotiations
C. It is a form of interaction among ASEAN members that is informal,
cooperative and respects cultural norms.
D. Never taking recourse to the judicial route to resolve commercial conflicts
between nations
User Answer :
Correct Answer : C
Answer Justification :

Learning: It is a working process or style that is informal and personal.

Policymakers constantly utilize compromise, consensus, and consultation in the


informal decision-making process. Above all it prioritizes a consensus-based, nonconflictual way of addressing problems.

SI

Quiet diplomacy allows ASEAN leaders to communicate without bringing the


discussions into the public view. Members avoid embarrassment that may lead to
further conflict.

IN

Q Source: Revision Previous Tests: 12th NCERT: Contemporary World Politics

33 Consider the following statements.


1. The governor can reserve certain types of bills passed by the state legislature for the
consideration of the President.
2. Bills on certain matters enumerated in the State List can be introduced in the state
legislature only with the previous sanction of the president.
3. The President can direct the states to reserve money bills and other financial bills
passed by the state legislature for his consideration during a financial emergency.
Select the correct answer using the codes below.
A. 1 and 2 only

(C) Insights Active Learning. | All rights reserved.

www.insightsias.com

22

TEST - 15

IA
S

B. 1 and 3 only
C. 2 and 3 only
D. 1, 2 and 3
User Answer :
Correct Answer : D
Answer Justification :
Learning: Besides the Parliament's power to legislate directly on the state subjects
under the exceptional situations, the Constitution empowers the Centre to exercise
control over the state's legislative matters in the above mentioned ways.
For example, the bills imposing restrictions on the freedom of trade and commerce
can be introduced only after Presidential assent.

TS

Q Source: Chapter 14: Indian Polity: M Laxmikanth

34 The Strait of Gibraltar is a narrow strait that connects


A. Atlantic Ocean to the Mediterranean Sea
B. Red Sea to Caspian Sea
C. Mediterranean Sea to Black Sea
D. Red Sea to Atlantic Ocean
User Answer :
Correct Answer : A
Answer Justification :

SI

Learning: It separates Gibraltar and Peninsular Spain in Europe from Morocco and
Ceuta (Spain) in Africa.

IN

The Strait has been identified as an Important Bird Area by BirdLife International
because hundreds of thousands of seabirds use it every year to pass between the
Mediterranean and the Atlantic.
The direct linkage of Atlantic Ocean to Mediterranean Sea creates certain unique
flow and wave patterns. You can read about them here.
https://en.wikipedia.org/wiki/Strait_of_Gibraltar#Special_flow_and_wave_patterns
Q Source: Revision Previous Tests (UPSC CSP) + World Geography questions +
Important Biodiversity Regions

(C) Insights Active Learning. | All rights reserved.

www.insightsias.com

23

TEST - 15

IA
S

35 Vehicles carrying inflammable material usually have metal ropes touching the ground
during motion. This is because
A. It helps to ignite engine spark easily in such heavily loaded vehicles
B. Velocity of the vehicle can be controlled this way.
C. The metal rope conducts the charge produced by air-friction in the vehicle
D. The vehicle needs to be insulated against lightning.
User Answer :
Correct Answer : C
Answer Justification :
Justification: When a vehicle moves, the body also gets charged due to air-friction
along with the tyres.

TS

If vehicle carries inflammable material, the charge accumulated on the body may
cause fire.
To avoid this, the accumulated charge is conducted to earth through the metallic
ropes.

Q Source: Revision Previous Tests: UPSC CAPF questions

IN

SI

36 New Public Management philosophy stands for


1. Complete privatization of commercial government undertakings
2. Decentralizing working units
3. Establishing short-term labour contracts instead of long-term agreements
Select the correct answer using the codes below.
A. 1 and 2 only
B. 2 and 3 only
C. 1 and 3 only
D. 1, 2 and 3
User Answer :
Correct Answer : B
Answer Justification :
Justification: Statement 1: NPM emphasizes the concept that ideas used in the
private sector must be successful in the public sector. But, it does not advocate
complete privatization of governmental functions.
Statement 2: This point suggest that it is more appropriate to shift from a unified
management system to a decentralized system in which managers gain flexibility
and are not limited to agency restrictions.

(C) Insights Active Learning. | All rights reserved.

www.insightsias.com

24

TEST - 15

Statement 3: It focuses on the necessity to establish short-term labor contracts,


develop corporate plans, performance agreements and mission statements.
Q Source: Revision Previous Tests: UPSC CAPF questions

TS

IA
S

37 Consider the following statements.


1. State Legislatures can make laws on matters enumerated in the Residuary list in
special circumstances.
2. State Legislatures cannot make laws on those matters enumerated in the Concurrent
List on which a Central legislation already exists.
Which of the above is/are correct?
A. 1 only
B. 2 only
C. Both 1 and 2
D. None
User Answer :
Correct Answer : D
Answer Justification :

Justification: Statement 1: The Parliament has exclusive powers to make laws with
respect to any of the matters enumerated in the Union List and Residuary list.

SI

Statement 2: Both, the Parliament and state legislature can make laws with respect
to any of the matters enumerated in the Concurrent List even if a Central law exists.
But, laws made by state should not contravene Central laws.
Q Source: Chapter 14: Indian Polity: M Laxmikanth

IN

38 Ricardian equivalence in economics refers to the notion that


A. Taxation and borrowing are equivalent means of financing expenditure
B. Fiscal deficit is an unsustainable exercise.
C. Investments and savings must be equal in a financial year in a closed
economy.
D. A country cannot maintain balance of payments if it wishes to lead in
international trade
User Answer :
Correct Answer : A
Answer Justification :
Learning: As per Ricardo, when the government increases spending by borrowing

(C) Insights Active Learning. | All rights reserved.

www.insightsias.com

25

TEST - 15

today, which will be repaid by taxes in the future, it will have the same impact on
the economy as an increase in government expenditure that is financed by a tax
increase today.

IA
S

It has often been argued that 'debt does not matter because we owe it to ourselves'.
This is because although there is a transfer of resources between generations,
purchasing power remains within the nation.
However, any debt that is owed to foreigners involves a burden since we have to
send goods abroad corresponding to the interest payments.
Q Source: Chapter on Public Finance: 12th NCERT Macroeconomics:

SI

TS

39 According to Buddhist philosophy


1. There is nothing permanent in the world.
2. Sorrow is intrinsic to human existence.
3. Either extreme penance or extreme self-indulgence will open the doors to salvation.
4. Human beings are unique in having a soul unlike other living beings.
Select the correct answer using the codes below.
A. 1 and 3 only
B. 2, 3 and 4 only
C. 1 and 2 only
D. 1, 3 and 4 only
User Answer :
Correct Answer : C
Answer Justification :

IN

Justification: Statement 1 and 4: According to Buddhist philosophy, the world is


transient (anicca) and constantly changing; it is also soulless (anatta) as there is
nothing permanent or eternal in it.
Statement 2 and 3: Within this transient world, sorrow is intrinsic to human
existence. It is by following the path of moderation between severe penance and
self-indulgence that human beings can rise above these worldly troubles.
Q Source: Revision of previous tests: Section on Buddhism: TamilNadu 11th
Standard History Textbook

40 Which of the following is/are the federal features of the Constitution?


1. The Constitution is written and not easily amendable.

(C) Insights Active Learning. | All rights reserved.

www.insightsias.com

26

TEST - 15

IA
S

2. State governments derive authority from the Centre.


3. Equal representation of all states in Rajya Sabha
Select the correct answer using the codes below.
A. 1 only
B. 2 and 3 only
C. 1 and 3 only
D. 1 and 2 only
User Answer :
Correct Answer : A
Answer Justification :

TS

Learning: The division of powers established by the Constitution as well as the


supremacy of the Constitution can be maintained only if the method of its
amendment is rigid. Hence, the Constitution is rigid to the extent that those
provisions which are concerned with the federal structure. So, 1 is correct.
State governments derive their authority from the constitution, not the Centre. So, 2
is wrong.

The states are given representation in the Rajya Sabha on the basis of population.
Hence, the membership varies from 1 to 31. So, 3 is wrong.

Q Source: Chapter 13: Indian Polity: M Laxmikanth

IN

SI

41 Which of the following is/are NOT mature Harappan sites?


1. Balakot
2. Rangpur
3. Ganweriwala
4. Digona
Select the correct answer using the codes below.
A. 1 and 2 only
B. 4 only
C. 3 and 4 only
D. 1, 2 and 3 only
User Answer :
Correct Answer : B
Answer Justification :
Learning:The image has been lifted from the Q source.
If you are unable to see the image, refer to the Q source.

(C) Insights Active Learning. | All rights reserved.

www.insightsias.com

27

TS

IA
S

TEST - 15

Q Source:Revision Previous Tests Syllabus: Chapter 1: 12th NCERT: Part-I

IN

SI

42 Very small animals are rarely found in the Polar Regions. Which of the following can
be the most appropriate reason for this?
A. Small animals have a larger surface area relative to their volume and lose
body heat very fast.
B. Very Small animals do not perform any metabolic activity which is necessary
to survive in the Polar Regions.
C. A large number of predators in Polar Regions prey on small animals.
D. All of (a), (b) and (c)
User Answer :
Correct Answer : A
Answer Justification :
Justification: Thermoregulation is energetically expensive for many organisms.
This is particularly true for small animals like shrews and humming birds.
Heat loss or heat gain is a function of surface area. Since small animals have a larger
surface area relative to their volume, they tend to lose body heat very fast when it is
cold outside; then they have to expend much energy to generate body heat through
metabolism.

(C) Insights Active Learning. | All rights reserved.

www.insightsias.com

28

TEST - 15

This is the main reason why very small animals are rarely found in Polar Regions.
Q Source: Revision Previous Tests Syllabus: 12th NCERT: Biology

TS

IA
S

43 Which of the following show the implementation of Directive Principles of State


Policy?
1. Establishment of Central Administrative Tribunal
2. Enactment of Legal Services Authorities Act
3. 73rd Amendment Act to the Constitution
4. Implementation of land reforms
Select the correct answer using the codes below.
A. 1, 3 and 4 only
B. 2, 3 and 4 only
C. 1 and 4 only
D. 1, 2, 3 and 4
User Answer :
Correct Answer : B
Answer Justification :

Learning: Statement 1: It addresses the service related grievances of central


government employees. It does not implement any DPSP.

SI

Statement 2: The Legal Services Authorities Act (1987) has established a nationwide network to provide free and competent legal aid to the poor and to organise lok
adalats for promoting equal justice.

IN

Statement 3: Three-tier panchayati raj system (at village, taluka and zila levels) has
been introduced to translate into reality Gandhiji's dream of every village being a
republic.
Statement 4: It reduces inequality and leads to welfare of weaker sections of the
population.
Q Source: Chapter 8: Indian Polity: M Laxmikanth

44 Which of the following features of human body prevent entry or growth of harmful
microbes in the body?
1. Skin
2. Acid in the stomach
3. Saliva

(C) Insights Active Learning. | All rights reserved.

www.insightsias.com

29

4. Eye tears
5. Mucus coating lining the respiratory tract
Select the correct answer using the codes below.
A. 1, 2 and 5 only
B. 4 and 5 only
C. 1, 3 and 4 only
D. 1, 2, 3, 4 and 5
User Answer :
Correct Answer : D
Answer Justification :

IA
S

TEST - 15

TS

Learning: Innate immunity is non-specific type of defence, which is present at the


time of birth. This is accomplished by providing different types of barriers to the
entry of the foreign agents into our body.
Innate immunity consists of four types of barriers.
These are -

SI

Physical barriers: Skin on our body is the main barrier which prevents entry
of the micro-organisms. Mucus coating of the epithelium lining the
respiratory, gastrointestinal and urogenital tracts also help in trapping
microbes entering our body.
Physiological barriers: Acid in the stomach, saliva in the mouth, tears from
eyes-all prevent microbial growth.
Cellular barriers: Certain types of leukocytes (WBC) of our body prevent
microbial growth.

IN

Q Source: Revision Previous Tests Syllabus: 12th NCERT Biology

45 The Parliament cannot amend those provisions which form the 'basic structure' of the
Constitution. This was ruled by the Supreme Court in the
A. Olga Tellis Case Vs. State of Maharashtra
B. Bhikaji Vs. Union of India case
C. Kesavananda Bharati vs. State of Kerala
D. Shankari Prasad Case
User Answer :
Correct Answer : C
Answer Justification :
Learning: Article 368 of the Constitution deals with the powers of Parliament to

(C) Insights Active Learning. | All rights reserved.

www.insightsias.com

30

TEST - 15

amend the Constitution and its procedure.


It states that the Parliament may, in exercise of its constituent power, amend by way
of addition, variation or repeal any provision of the Constitution in accordance with
the procedure laid down for the purpose.

IA
S

But, if the amendment alters the basic structure, for e.g. Judicial review,
Parliamentary democracy, Rule of Law, Secularism etc. then the amendment can be
declared ultra vires by the SC.
Q Source: Chapter 10: Indian Polity: M Laxmikanth

SI

TS

46 The Tamil society witnessed a great change during the Pallava period. Which of the
following about the Pallava period is correct?
1. The caste system became rigid.
2. The period witnessed the rise of Saivism and Vaishnavism.
3. Buddhism and Jainism declined.
Select the correct answer using the codes below.
A. 1 and 2 only
B. 2 and 3 only
C. 1 and 3 only
D. 1, 2 and 3
User Answer :
Correct Answer : D
Answer Justification :

IN

Learning: The Brahmins occupied a high place in the society. They were given
land-grants by the kings and nobles. They were also given the responsibility of
looking after the temples.
The Saiva Nayanmars and the Vaishnava Alwars contributed to the growth of
Saivism and Vaishnavism. This is known as the Bakthi Movement.
They composed their hymns in the Tamil language. These hymns revealed the
importance of devotion or Bakthi. The construction of temples by the Pallava kings
paved the way for the spread of these two religions.
Q Source:Revision Previous Tests Syllabus: 11th TamilNadu History Textbook

47 Consider the following statements. Assertion (A): Marine sector contributes more to

(C) Insights Active Learning. | All rights reserved.

www.insightsias.com

31

TEST - 15

IA
S

fishery production than inland sources. Reason (R): India has a long coastline with a large
coastal community involved in fishing. In the context of the above, which of these is
correct?
A. A is correct, and R is an appropriate explanation of A.
B. A is correct, but R is not an appropriate explanation of A.
C. A is incorrect, but R is correct.
D. Both A and R are correct independently.
User Answer :
Correct Answer : C
Answer Justification :

TS

Justification: The total fish production during 2013-14(Provisional) is registered


9.58 million metric tonnes, with a contribution of 6.14 million metric tonnes from
inland sector and 3.44 million metric tonnes from marine sector. So, A is wrong.
R is correct as the fisheries sector is a source of livelihood for over 14.49 million
people engaged fully, partially or in subsidiary activities related to the sector.

Q Source: Revision Previous Tests Syllabus: 11th NCERT: Indian Economic


Development

IN

SI

48 Which of the following is/are the implications of promoting organic farming?


1. International demand for organic food can be fulfilled.
2. Organic farming is more labour intensive and suits a labour surplus economy India.
3. It will give a fillip to the Genetic Engineering sector.
4. It will make farming expensive as it requires costlier inputs.
Select the correct answer using the codes below.
A. 1 and 2 only
B. 1 and 4 only
C. 2 and 3 only
D. 1, 2 and 3
User Answer :
Correct Answer : A
Answer Justification :
Justification:Statement 1: Organic agriculture also generates incomes through
international exports as the demand for organically grown crops is on a rise. Studies
across countries have shown that organically grown food has more nutritional value
than chemical farming thus providing us with healthy foods
Statement 2: Since organic farming requires more labour input than conventional

(C) Insights Active Learning. | All rights reserved.

www.insightsias.com

32

TEST - 15

farming, India will find organic farming an attractive proposition being a labour
surplus economy.
Statement 3: It does not use GM crops.

IA
S

Statement 4: Organic agriculture offers a means to substitute costlier agricultural


inputs (such as HYV seeds, chemical fertilisers, pesticides etc.) with locally
produced organic inputs that are cheaper and thereby generate good returns on
investment.
Q Source: Revision Previous Tests Syllabus: 11th NCERT: Indian Economic
Development

TS

49 To get just and humane conditions for work and maternity relief is a
A. Fundamental Right
B. Legal Right
C. Directive Principle of State Policy
D. Executive Decree
User Answer :
Correct Answer : C
Answer Justification :

Learning: Some of the labour related DPSPs are:

IN

SI

To make provision for just and humane conditions for work and maternity
relief (Article 42).
To secure a living wage, a decent standard of life and social and cultural
opportunities for all workers (Article 43).
To take steps to secure the participation of workers in the management of
industries (Article 43 A).

Q Source:Chapter 8: Indian Polity: M Laxmikanth

50 Which of the following Directive Principles did NOT exist in the unamended original
constitution?
1. To protect and improve the environment and to safeguard forests and wild life
2. To secure for all citizens a uniform civil code throughout the country
3. To secure the right to adequate means of livelihood for all citizens
Select the correct answer using the codes below.
A. 1 and 2 only

(C) Insights Active Learning. | All rights reserved.

www.insightsias.com

33

TEST - 15

IA
S

B. 2 and 3 only
C. 1 only
D. 1, 2 and 3
User Answer :
Correct Answer : C
Answer Justification :
Learning: The 42nd Amendment Act of 1976 added four new Directive Principles
to the original list. They require the State:

TS

To secure opportunities for healthy development of children (Article 39).


To promote equal justice and to provide free legal aid to the poor (Article 39
A).
To take steps to secure the participation of workers in the management of
industries (Article
43 A).
To protect and improve the environment and to safeguard forests and wild
life (Article 48 A).
The 97th Amendment Act of 2011 added a new Directive Principle relating to
co-operative societies. It requires the state to promote voluntary formation,
autonomous functioning, democratic control and professional management of
co-operative societies (Article 43B).

Q Source: Chapter 8: Indian Polity: M Laxmikanth

IN

SI

51 The Wholesale Price Index (WPI) is released by


A. Department of Finance, Ministry of Finance
B. Central Statistical Organization, Ministry of Statistics and Programme
Implementation
C. Office of the Economic Adviser, Department of Industrial Policy &
Promotion
D. Office of the Governor, Reserve Bank of India
User Answer :
Correct Answer : C
Answer Justification :
Learning:WPI is an important statistical indicator, as various policy decisions of
the
Government, like inflation management, monitoring of prices of essential
commodities etc., are based on it.

(C) Insights Active Learning. | All rights reserved.

www.insightsias.com

34

TEST - 15

It is one of the key variables for monetary policy changes by the Reserve Bank of
India. In addition to its role as a policy variable, WPI is also used by various
departments for arriving at the escalation costs of various contracts.
Headline inflation in India is measured in terms of Wholesale Price Index (WPI).

TS

52 The Vesara style reached its culmination under


A. Rashtrakutas and Hoysalas
B. Chalukyas
C. Satvahanas
D. Imperial Cholas
User Answer :
Correct Answer : A
Answer Justification :

IA
S

Q Source: Chapter 7: Indian Economy - Ramesh Singh

Learning: The Chalukyas were great patrons of art. They developed the vesara style
in the building of structural temples.
The structural temples of the Chalukyas exist at Aihole, Badami and Pattadakal.

Q Source: Revision Previous Tests Syllabus: 11th TamilNadu History Textbook

IN

SI

53 The Parliament can enact laws to implement which of the following fundamental duties
under Part IVA of the Constitution?
1. Penalty for not filing tax returns
2. Penalty for not voting in elections
3. Punishment for spreading communal hatred
Select the correct answer using the codes below.
A. 1 and 2 only
B. 3 only
C. 1 and 3 only
D. 1, 2 and 3
User Answer :
Correct Answer : B
Answer Justification :
Justification: Filing tax returns and voting in elections are not fundamental duties.
So, 1 and 2 will be wrong.

(C) Insights Active Learning. | All rights reserved.

www.insightsias.com

35

TEST - 15

It is a fundamental duty to promote harmony and the spirit of common brotherhood


amongst all the people of India transcending religious boundaries. Law can be
enacted to make the provision legally effective.
Q Source:Chapter 9: Indian Polity: M Laxmikanth

TS

IA
S

54 Which of these is/are NOT Fundamental Duties of an Indian citizen?


1. To properly nourish his/her children.
2. To safeguard national property.
3. To defend the country and render national service when called upon.
Select the correct answer using the codes below.
A. 1 and 2 only
B. 3 only
C. 1 only
D. 2 and 3 only
User Answer :
Correct Answer : C
Answer Justification :

Justification:To provide opportunities for education to his child or ward between


the age of six and fourteen years is a fundamental duty. Nourishment, health,
happiness etc don't come in it. So, 1 is wrong.

SI

If there was an option like respect the elderly, you can tickmark it right, since
valuing and preserving the rich heritage of the country's composite culture is a
fundamental duty.
Q Source: Chapter 9: Indian Polity: M Laxmikanth

IN

55 The Union Cabinet Committee on Economic Affairs (CCEA) has approved Pradhan
Mantri Ujjwala Yojana (PMUY) for
A. Providing free of cost LPG connections to women from BPL Households.
B. Giving subsidized LED lights to power consumers
C. Providing scholarships to BPL students for higher education
D. Giving free electricity connections in Naxal affected blocks
User Answer :
Correct Answer : A
Answer Justification :
Learning: The PMUY seeks to empowering women and protecting their health by

(C) Insights Active Learning. | All rights reserved.

www.insightsias.com

36

TEST - 15

shifting them from traditional cooking based on unclean cooking fuels or on fossil
fuels that have serious health hazards to clean cooking gas.

IA
S

Financial support of 1600 rupees for each LPG connection will be provided to the
BPL households. The identification of eligible BPL families will be made in
consultation with the State Governments and the Union Territories. It will be
implemented over three years' time frame namely in the FY 2016-17, 2017-18 and
2018-19.
Q Source:
http://www.newindianexpress.com/states/odisha/Ujjwala-Yojana-to-Put-Odisha-onSticky-Wicket/2016/03/12/article3322905.ece

TS

56 Consider the following statements.Assertion (A): The president must give her assent to
a Constitutional amendment bill passed by due process.Reason (R): The Constitution
makers did not give the office of the President any responsibility with regard to protecting
the constitution.In the context of the above, which of these is correct?
A. A is correct, and R is an appropriate explanation of A.
B. A is correct, but R is not an appropriate explanation of A.
C. A is incorrect, but R is correct.
D. A is correct, but R is incorrect.
User Answer :
Correct Answer : D
Answer Justification :

SI

Justification:The president must give his assent to the bill. He can neither withhold
his assent to the bill nor return the bill for reconsideration of the Parliament.

IN

The 24th Constitutional Amendment Act of 1971 made it obligatory for the
President to give his assent to a constitutional Amendment Bill.
Before this he could treat a constitutional amendment bill just as an ordinary bill and
could use his veto powers. So, R is wrong.
Q Source: Chapter 10: Indian Polity: M Laxmikanth

57 Which of the following provisions of the Constitution can be amended by a simple


majority in the Parliament?
1. Conferment of more jurisdiction on the Supreme Court
2. Delimitation of constituencies

(C) Insights Active Learning. | All rights reserved.

www.insightsias.com

37

3. Directive Principles of State Policy


4. Election of the President and its manner
Select the correct answer using the codes below.
A. 1 and 2 only
B. 3 and 4 only
C. 1 only
D. 1, 2, 3 and 4
User Answer :
Correct Answer : A
Answer Justification :

IA
S

TEST - 15

Justification: Statement 3 requires special majority.

Statement 4 requires Special Majority of Parliament and Consent of States.

TS

Other important provisions that can be changed by simple majority of Parliament


are:

Use of official language.


Citizenship-acquisition and termination.
Elections to Parliament and state legislatures.
Delimitation of constituencies.
Union territories.
Fifth Schedule-administration of scheduled areas and scheduled tribes.
Sixth Schedule-administration of tribal areas.

SI

Q Source:Chapter 10: Indian Polity: M Laxmikanth

IN

58 The lower house of the Parliament, Lok Sabha, can be dissolved by


A. The President on recommendation of the Prime Minister
B. The Prime Minister on the advice of the Council of Ministers
C. The Supreme Court on an inquiry conducted by a Parliamentary Committee
D. The Speaker on the recommendations of a Joint Committee of both houses
User Answer :
Correct Answer : A
Answer Justification :
Learning:The prime minister can advise the President to dissolve the Lok Sabha
before the expiry of its term and hold fresh elections.
This means that the executive enjoys the right to get the legislature dissolved in a

(C) Insights Active Learning. | All rights reserved.

www.insightsias.com

38

TEST - 15

parliamentary system.
Q Source:Chapter 12: Indian Polity: M Laxmikanth

IA
S

59 Sri Lanka is separated from India by a narrow channel of sea formed by the
A. Gulf of Mannar
B. Palk Strait
C. Kandy Isthmus
D. Hambantota Bridge
User Answer :
Correct Answer : B
Answer Justification :

TS

Learning: It is a strait between Tamil Nadu and Mannar district of the Northern
Province of Sri Lanka.
It connects the Bay of Bengal in the northeast with the Palk Bay and thence with the
Gulf of Mannar in the southwest.

It is studded at its southern end with a chain of low islands and reef shoals that are
collectively called Adam's Bridge, since ages it is popularly known in Hindu
Mythology as "Ram Setu".
Q Source: Chapter 1: India Yearbook 2016

IN

SI

60 'Inflationary Gap' refers to the


A. Excess demand of goods over production in the economy
B. Large difference in inflation in the preceding and succeeding year
C. Inflation that does not trigger an increase in GDP
D. Distance between the current level of real GDP and full employment real
GDP
User Answer :
Correct Answer : D
Answer Justification :
Learning: Real GDP is nominal GDP adjusted for inflation.
The inflationary gap is so named because the relative increase in real GDP causes an
economy to increase its consumption, which causes prices to rise in the long run.

(C) Insights Active Learning. | All rights reserved.

www.insightsias.com

39

TEST - 15

The main cause of the gap is considered to be expansionary monetary policies


carried out by the government.

IA
S

An inflationary gap is a signal that the economy is in the boom part of the trade
cycle, resources are being used over their capacity, factories are operating with
increasing average costs; wage rates increase because labour is used beyond normal
hours at overtime pay rates.
Q Source: Chapter 7: Indian Economy - Ramesh Singh

TS

61 The principle of collective responsibility is the bedrock principle of parliamentary


government. The principle implies that the
1. Lok Sabha can remove the council of ministers from office by passing a vote of no
confidence.
2. The Council of Ministers as a whole is bound by consensus.
Which of the above is/are correct?
A. 1 only
B. 2 only
C. Both 1 and 2
D. None
User Answer :
Correct Answer : C
Answer Justification :

SI

Learning: The ministers are collectively responsible to the Parliament in general


and to the Lok Sabha in particular (Article 75).

IN

They act as a team, and swim and sink together. The principle of collective
responsibility implies that the Lok Sabha can remove the ministry (i.e., council of
ministers headed by the prime minister) from office by passing a vote of no
confidence.
Members of the council of ministers are bound by consensus. The government
cannot have two opinions on the same issue.
Q Source: Chapter 12: Indian Polity: M Laxmikanth

62 Consider the following with reference to the Gender related data as recorded by the
Census 2011.
1. It has recorded the highest ever sex ratio in India since independence.

(C) Insights Active Learning. | All rights reserved.

www.insightsias.com

40

TEST - 15

IA
S

2. More number of females have become literate than males since Census 2001.
Which of the above is/are correct?
A. 1 only
B. 2 only
C. Both 1 and 2
D. None
User Answer :
Correct Answer : B
Answer Justification :

TS

Justification: Statement 1: Overall sex ratio at the national level increased by 7


points since census 2001 to reach 943 at census 2011. This is the highest sex ratio
recorded since census 1961, not since independence. It is lower than the sex ratio in
1951 (it was 946).
Statement 2: Literacy rate has gone up from 64.8 per cent in 2001 to 73.0 per cent
showing an increase of 8.2 percentage points. It is encouraging to note that out of a
total of 202,810,720 literates added during the decade, females 104,660,657
outnumber males 98,150,063.

Q Source: Chapter 1: India Yearbook 2016

IN

SI

63 The parliamentary system of government in India is largely based on the British


parliamentary system. However the Indian system differs from the British in which of the
following?
1. India is a Republic whereas Britain is a constitutional Monarchy.
2. Citizens other than MPs cannot be appointed as Ministers in India unlike in Britain.
3. In Britain the prime minister should be a member of the Lower House unlike in
India where he can be a member of any house.
Select the correct answer using the codes below.
A. 1 and 2 only
B. 2 and 3 only
C. 1 and 3 only
D. 1, 2 and 3 only
User Answer :
Correct Answer : C
Answer Justification :
Justification: In Britain, the prime minister should be a member of the Lower
House (House of Commons) of the Parliament. In India, the prime minister may be a

(C) Insights Active Learning. | All rights reserved.

www.insightsias.com

41

TEST - 15

member of any of the two Houses of Parliament


Usually, the members of Parliament alone are appointed as ministers in Britain. In
India, a person who is not a member of Parliament can also be appointed as minister,
but for a maximum period of six months.

IA
S

Q Source: Chapter 12: Indian Polity: M Laxmikanth

SI

TS

64 Many early sculptors did not show the Buddha in human form - instead, they showed
his presence through symbols. Which of the following symbols have been used to represent
the Buddha and his teachings?
1. Empty Seat
2. Divine Light
3. Wheel
4. The Bodhi Tree
5. Endless Knots
6. Plain white cloth
Select the correct answer using the codes below.
A. 1, 2, 4 and 6 only
B. 2, 3 and 6 only
C. 1, 3, 4 and 5 only
D. 1, 2, 3, 4, 5 and 6
User Answer :
Correct Answer : C
Answer Justification :

IN

Learning: Mahayana and Vajrayana Buddhist art frequently makes use of a


particular set of eight auspicious symbols, ashtamangala, in domestic and public art.
These symbols have spread with Buddhism to the art of many cultures, including
Indian, Tibetan, Nepalese, and Chinese art.
These symbols are:
Lotus flower. Representing purity and enlightenment.
Endless knot, or, the Mandala. Representing eternal harmony.
Golden Fish pair. Representing conjugal happiness and freedom.
Victory Banner. Representing a victorious battle.
Wheel of Dharma or Chamaru in Nepali Buddhism. Representing knowledge.
Treasure Vase. Representing inexhaustible treasure and wealth.
Parasol. Representing the crown, and protection from the elements.
Conch shell. Representing the thoughts of the Buddha.

(C) Insights Active Learning. | All rights reserved.

www.insightsias.com

42

TEST - 15

Q Source: Revision of previous tests: Section on Buddhism: TamilNadu 11th


Standard History Textbook

TS

IA
S

65 Which of the following trends in the working of Indian political system reflects its
federal spirit?
1. Territorial disputes between states
2. Demand for creation of new states
3. Demand of the states for more financial grants from the Centre to meet their
developmental needs
4. Emergence of Regional parties and their contribution in national politics
5. Opposition of the State to the Goods and Services Tax (GST)
Select the correct answer using the codes below.
A. 1, 2 and 3 only
B. 2, 4 and 5 only
C. 3, 4 and 5 only
D. 1, 2, 3, 4 and 5
User Answer :
Correct Answer : D
Answer Justification :

Learning: Although the Constitution of India has created a strong Central


government, it has not made the state governments weak and has not reduced them
to the level of administrative agencies for the execution of policies of the Central
government.

SI

Indian federation has been described as as "a new kind of federation to meet India's
peculiar needs".

IN

It is clearly seen from the above examples. This implies that the state governments
are autonomous units working under Constitutional framework in tandem with the
Centre to achieve common goals.
Q Source: Chapter 13: Indian Polity: M Laxmikanth

66 The National Air Quality Index takes into account which of the following air pollutants?
1. Particulate Matter
2. Asbestos
3. Ozone
4. Lead
5. Carbon dioxide

(C) Insights Active Learning. | All rights reserved.

www.insightsias.com

43

Select the correct answer using the codes below.


A. 1, 3 and 4 only
B. 2, 3 and 4 only
C. 1 and 5 only
D. 1, 2, 3, 4 and 5
User Answer :
Correct Answer : A
Answer Justification :

IA
S

TEST - 15

Learning: The National AQI is published for every month by CPCB along with a
numerical value and a colour code which helps in comparing air pollution levels in
cities.

TS

It is determined on the basis of concentration of 8 pollutants, including Particulate


Matter (PM 2.5, PM 10), sulphur dioxide (SO2), nitrogen dioxide (NO2), carbon
monoxide (CO), ozone (O3), ammonia (NH3) and lead (Pb).

The colour categories are classified into 6 categories depending upon numerical
value as Good (0-50), Satisfactory (51-100), moderately polluted (101-200), Poor
(201-300), Very poor (301-400) and Severe (401-500).
Q Source: Previous year Current affairs

http://pib.nic.in/newsite/PrintRelease.aspx?relid=110654

IN

SI

67 The Constitution empowers the Parliament to make laws on any matter enumerated in
the State List under which of the following extraordinary circumstances?
1. When the President exhorts the Council of Ministers to do so in national interest
2. When the lower house passes a resolution to this effect by special majority
3. When a National Emergency is in operation
4. When the National Security Council recommends so in the interest of national
security
Select the correct answer using the codes below.
A. 1 and 2 only
B. 3 only
C. 3 and 4 only
D. 1, 2, 3 and 4
User Answer :
Correct Answer : B
Answer Justification :

(C) Insights Active Learning. | All rights reserved.

www.insightsias.com

44

TEST - 15

Learning: If the Rajya Sabha declares that it is necessary in the national interest
that Parliament should make laws on a matter in the State List, then the Parliament
becomes competent to make laws on that matter. This cannot be declared by the
President, Lok Sabha or the National Security Council. So, all statements except 3
are wrong.

IA
S

The Parliament can make laws on any matter in the State List for implementing the
international treaties, agreements or conventions.
Q Source:Chapter 14: Indian Polity: M Laxmikanth

TS

68 Consider the following statements about United Nations (UN) peacekeeping operations.
1. UN maintains an independent peacekeeping army funded by contributions from
member states.
2. UN Security Council authorizes peacekeeping operations.
3. Regional organizations can be authorized to undertake peacekeeping operations.
Select the correct answer using the codes below.
A. 1 and 2 only
B. 2 and 3 only
C. 2 only
D. 1 and 3 only
User Answer :
Correct Answer : B
Answer Justification :

IN

SI

Justification:Statement 1 and 3 : Most of these operations are established and


implemented by the United Nations itself, with troops serving under UN operational
control.
In these cases, peacekeepers remain members of their respective armed
forces, and do not constitute an independent "UN army," as the UN does not
have such a force.
In cases where direct UN involvement is not considered appropriate or
feasible, the Council authorizes regional organizations such as the NATO or
coalitions of willing countries to undertake peacekeeping or peaceenforcement tasks.

Statement 2: The UN Charter gives the UNSC the power and responsibility to take
collective action to maintain international peace and security. For this reason, the
international community usually looks to the UNSC to authorize peacekeeping
operations through Chapter VI authorizations.

(C) Insights Active Learning. | All rights reserved.

www.insightsias.com

45

TEST - 15

Q Source: Current affairs + Revision Previous Tests: 12th NCERT: Contemporary


World Politics

TS

IA
S

69 Inflation in India is measured 'point-to-point'. It can mean that


A. Reference dates for calculating inflation are from the preceding and
succeeding month of the same year
B. Reference dates for calculating inflation are from the preceding and
succeeding quarter of the same year
C. The same date is taken as reference for calculating inflation from the
preceding and succeeding years
D. All of (a), (b), (c)
User Answer :
Correct Answer : C
Answer Justification :

Justification: It means that the reference dates for the annual inflation is January 1
to January 1 of two consecutive years (not for January 1 to December 31 of the
concerned year).

Similarly, the weekly rate of inflation is the change in one week reference being the
two consecutive last days of the week (i.e., 5 p.m. of two Fridays in India).

Q Source:Chapter 7: Indian Economy - Ramesh Singh

IN

SI

70 Which of the following are extra-constitutional devices to promote cooperation and


coordination between the Centre and the states?
1. Zonal Councils
2. Inter-State Councils
3. North-Eastern Council
4. Central Council of Indian Medicine
Select the correct answer using the codes below.
A. 1 and 3 only
B. 2 and 4 only
C. 1, 3 and 4 only
D. 1, 2, 3 and 4 only
User Answer :
Correct Answer : C
Answer Justification :
Learning: Subject specific Inter-State Councils set up under Article 263 of the

(C) Insights Active Learning. | All rights reserved.

www.insightsias.com

46

TEST - 15

Constitution of India
Central Council of Health
Central Council for Local Government and Urban Development
Regional Councils for Sales Tax and State Excise Duties

IA
S

Inter-State Coordination Mechanism set up outside the framework of Article 263 of


the Constitution of India

TS

Planning Commission of India (Now abolished)


National Development Council
National Integration Council
Central Advisory Board of Education
Central Council for Research in Ayurveda & Siddha
Central Council for Research in Homoeopathy
Central Council for Research in Yoga & Naturopathy
Labour Conference

Q Source: Chapter 14: Indian Polity: M Laxmikanth

IN

SI

71 From the various judgements of the Supreme Court, which of the following have
emerged as elements of 'basic structure' of the Constitution?
1. Separation of powers between the legislature, the executive and the judiciary
2. Harmony and balance between Fundamental Rights and Directive Principles
3. Supremacy of the Constitution
4. Principle of reasonableness
5. Welfare state and ideals of Socio-economic Justice
Select the correct answer using the codes below.
A. 1, 2 and 3 only
B. 1, 3 and 5 only
C. 2, 4 and 5 only
D. 1, 2, 3, 4 and 5
User Answer :
Correct Answer : D
Answer Justification :
Learning: Apart the above, these are the elements:
Sovereign, democratic and republican nature of the Indian polity
Secular character of the Constitution
Federal character of the Constitution

(C) Insights Active Learning. | All rights reserved.

www.insightsias.com

47

TEST - 15

IA
S

Unity and integrity of the nation


Judicial review
Freedom and dignity of the individual
Parliamentary system
Rule of law
Principle of equality
Free and fair elections
Independence of Judiciary
Limited power of Parliament to amend the Constitution
Effective access to justice
Powers of the Supreme Court under Articles 32, 136, 141 and 142
Q Source:Chapter 11: Indian Polity: M Laxmikanth

TS

72 Statutory grants given under Article 275 are


A. Grants given to states in need of financial assistance and not to every state
B. Grants made for any public purpose by the Centre to the States
C. Temporary grants given by the Centre to the states which the states must
repay in due time
D. Grants recommended by erstwhile Planning Commission
User Answer :
Correct Answer : A
Answer Justification :

SI

Learning: Article 275 empowers the Parliament to make grants to the states which
are in need of financial assistance and not to every state. Also, different sums may
be fixed for different states. These sums are charged on the Consolidated Fund of
India every year.

IN

Apart from this general provision, the Constitution also provides for specific grants
for promoting the welfare of the scheduled tribes in a state or for raising the level of
administration of the scheduled areas in a state including the State of Assam.
The statutory grants under Article 275 (both general and specific) are given to the
states on the recommendation of the Finance Commission.
Q Source:Chapter 14: Indian Polity: M Laxmikanth

73 Every year International Women's Day is being observed on 8th March across the globe
to highlight the achievements of women while calling for greater equality. What is the

(C) Insights Active Learning. | All rights reserved.

www.insightsias.com

48

TEST - 15

IA
S

theme for 2016 as declared by the United Nations?


A. Planet 50-50 by 2030: Step It Up for Gender Equality
B. World Free of Violence Against Women
C. Empower Rural Women, End Poverty and Hunger
D. Women in Decision-making
User Answer :
Correct Answer : A
Answer Justification :

Learning:Themes in options other than (a) are previous year themes.

The day is an official holiday in countries like Afghanistan, Angola, Armenia, Nepal
(women only) etc.

TS

The earliest celebration was held as a Socialist political event in 1909. It blended the
culture of many countries primarily in Europe, especially those in the Soviet Bloc.
Declared a national holiday in the Soviet Union in 1917, it spread to other nearby
countries. It is now celebrated in many Eastern countries.

Q Source:International Women's Day

IN

SI

74 The President can proclaim a national emergency when


A. Cabinet gives a written recommendation to the President
B. The Parliament passes a resolution by simple majority to this effect
C. The President is individually satisfied that it is in the best national interest
D. The National Security Council recommends it to the President
User Answer :
Correct Answer : A
Answer Justification :
Learning:The emergency can be declared only on the concurrence of the cabinet
and not merely even on the advice of the prime minister or Home Minister.
The proclamation of Emergency should be approved by both the Houses of
Parliament within one month from the date of its issue. So, (b) is wrong.
In 1975, the then Prime Minister, Indira Gandhi advised the president to proclaim
emergency without consulting her cabinet.
The cabinet was informed of the proclamation after it was made, as a fait accompli.

(C) Insights Active Learning. | All rights reserved.

www.insightsias.com

49

TEST - 15

The 44th Amendment Act of 1978 introduced this safeguard to eliminate any
possibility of the prime minister alone taking a decision in this regard.
Q Source:Chapter 16: Indian Polity: M Laxmikanth

TS

IA
S

75 During a national emergency


1. State governments are suspended and Centre takes the control of the states.
2. Parliament becomes empowered to make laws on any subject mentioned in the State
List.
Which of the above is/are correct?
A. 1 only
B. 2 only
C. Both 1 and 2
D. None
User Answer :
Correct Answer : B
Answer Justification :

Justification:Statement 1: The Centre becomes entitled to give executive directions


to a state on 'any' matter. Thus, the state governments are brought under the
complete control of the Centre, though they are not suspended.

Statement 2: Although the legislative power of a state legislature is not suspended, it


becomes subject to the overriding power of the Parliament.

SI

Thus, the normal distribution of the legislative powers between the Centre and states
is suspended, though the state legislatures are not suspended. In brief, the
Constitution becomes unitary rather than federal.

IN

Q Source:Chapter 16: Indian Polity: M Laxmikanth

76 Phillips Curve shows the trade-off between


A. GDP growth and population growth
B. Inflation and unemployment
C. Environmental degradation and economic inequality
D. Real GDP growth and inflation
User Answer :
Correct Answer : B
Answer Justification :

(C) Insights Active Learning. | All rights reserved.

www.insightsias.com

50

TEST - 15

Learning:As per the curve there is a 'trade off' between inflation and unemployment
i.e. an inverse relationship between them. The curve suggests that lower the
inflation, higher the unemployment and higher the inflation, lower the
unemployment.

IA
S

During 1960s, this idea was among the most important theories of the modern
economists.
However, economists later contested this curve and suggested some modifications to
it.
Q Source:Chapter 7: Indian Economy - Ramesh Singh

TS

77 The presidential proclamation imposing President's Rule is subject to judicial review


was declared by the Supreme Court in
A. D.C. Wadhwa Case
B. S.R. Bommai Case
C. Rameshwar Prasad Case
D. M.C. Mehta Case
User Answer :
Correct Answer : B
Answer Justification :

SI

Learning: The Supreme Court declared in D. C. Wadhwa v. State of Bihar (1986),


that it is unconstitutional to repromulgate ordinances (in certain conditions).
The case of M.C. Mehta v. Union of India originated in the aftermath of oleum gas
leak from Shriram Food and Fertilisers Ltd. complex at Delhi.

IN

Rameshwar Prasad case was related to dissolution of Bihar Assembly.


Q Source:Chapter 16: Indian Polity: M Laxmikanth

78 The song Vande Mataram, composed in Sanskrit by Bankimchandra Chatterji, was a


source of inspiration to the people in their struggle for freedom. It was first sung at
A. 1929 Purna Swaraj declaration session at Lahore
B. Calcutta during the mutiny of 1857
C. 1896 session of the Indian National Congress
D. Swadeshi movement 1905 after partition of Bengal
User Answer :

(C) Insights Active Learning. | All rights reserved.

www.insightsias.com

51

TEST - 15

Correct Answer : C
Answer Justification :
Learning:It has an equal status with Jana-gana-mana, our national anthem.

IA
S

Jana-gana-mana was first sung in 1911 at the Kolkata Session of the Indian National
Congress.
The English translation of Vande Matram stanzas was rendered by Sri Aurobindo.
Q Source:National Song: Chapter 2: India Yearbook 2016

TS

79 An unmanned spacecraft named ExoMars 2016 was launched jointly Europe and Russia
to
1. Search for biosignatures on Mars in past or present.
2. Study the outer atmosphere of Mars and its impact on earths long-term climate.
Which of the above is/are correct?
A. 1 only
B. 2 only
C. Both 1 and 2
D. None
User Answer :
Correct Answer : A
Answer Justification :

SI

Learning:Its primary Goal is to address the question of whether life has ever existed
on Mars.

IN

Its sole purpose is to search (trace) evidence of methane (CH4) and other
atmospheric gases that could be signatures of active biological or geological
processes on Mars.
For the first time it will provide new insights into the role of electric forces on dust
lifting, the trigger for dust storms on the Martian surface.
Q
Source:http://www.thehindu.com/todays-paper/tp-national/exomars-giant-nose-to-s
niff-out-life-on-mars/article8349939.ece

80 Consider the following about the missile Agni-I.

(C) Insights Active Learning. | All rights reserved.

www.insightsias.com

52

TEST - 15

IA
S

1. It is a nuclear capable surface-to-surface ballistic missile.


2. It can travel faster than the speed of sound.
3. Its sophisticated navigation system can navigate the missile to hit target accurately.
Select the correct answer using the codes below.
A. 1 and 2 only
B. 1 only
C. 2 and 3 only
D. 1, 2 and 3
User Answer :
Correct Answer : D
Answer Justification :

TS

Learning:Agni-I is the first missile of the Agni series launched in 1983 by the
DRDO as part of Integrated Guided Missile Development Programme (IGMDP) in
India.
Its testing is conducted as part of training exercise by the Strategic Forces Command
(SFC) of Indian Army.

It is powered by both solid and liquid propellants and can be fired from road and rail
mobile launchers.

The Agni I missile already has been inducted into armed forces and claimed to be a
part of the India's minimum credible deterrence under No first to use policy.

SI

Q
Source:http://www.thehindu.com/news/national/india-testfires-nuclearcapable-agnii
/article8351534.ece

IN

81 China's Supreme Court has decided to set up its own International Maritime Judicial
Centre (IMJC) to handle territorial disputes and protect its sea rights. However,
International maritime disputes between countries are usually brought before the
A. International Court of Justice (ICJ)
B. International maritime organisation (IMO)
C. United Nations Security Council (UNSC)
D. International Chamber of Shipping (ICM)
User Answer :
Correct Answer : A
Answer Justification :
Learning:Presently China is locked in disputes with its neighbours over claims in

(C) Insights Active Learning. | All rights reserved.

www.insightsias.com

53

TEST - 15

the resource-rich South China Sea.


The tensions have risen recently over China's aggressive land reclamation continues
to build artificial islands, airport runways and facilities on disputed reefs.

IA
S

This move will help China to bolster its claims in the disputed South and East China
seas and also help it become a maritime power.
Q
Source:http://www.business-standard.com/article/news-ians/china-to-build-internati
onal-maritime-judicial-centre-116031300070_1.html

TS

82 The Government of India is celebrating April 21 every year as 'Civil Services Day'. 21st
April has been chosen as date because
A. Sardar Vallabhbhai Patel addressed the first batch of Indian Administrative
Services (IAS) officers on this date.
B. Indian Covenanted Civil Services were created by the British on this date
C. First batch of Civil servants of Independent India graduated on this date.
D. Pandit Nehru chose this date based on the first International meeting of Civil
Servants.
User Answer :
Correct Answer : A
Answer Justification :

SI

Learning: On the occasion, civil servants are awarded by the Prime Minister for
their excellent work done in the field of public administration.

IN

It is celebrated as an occasion for the civil servants to rededicate themselves to the


cause of citizens and renew their commitment to public service and excellence in
work.
First such function was held in 2006.
Q Source:Chapter 3: India Yearbook 2016

83 Wholesale Price Index (WPI) indicates price rise in which of the following categories?
1. Fuel
2. Power
3. Gold and Silver

(C) Insights Active Learning. | All rights reserved.

www.insightsias.com

54

TEST - 15

IA
S

4. Some consumer durables


Select the correct answer using the codes below.
A. 1 and 2 only
B. 4 only
C. 1, 2 and 4 only
D. 1, 2, 3 and 4
User Answer :
Correct Answer : D
Answer Justification :

Justification:There are three major groups in WPI - Primary Articles, Fuel & Power
and Manufacture Products. Highest weightage has been accorded to the
manufactured products.

TS

Consumer durables are a part of the manufactured products category.


The WPI series base year, article composition and weightage has also been revised
few times.

The Office of the Economic Adviser undertook the work relating to revision of the
existing series of WPI, which not only addressed the issue of change in base year,
but also revised the entire commodity basket and the weighting diagram so as to
better reflect the price trends in economy.

SI

Q Source:Chapter 7: Indian Economy - Ramesh Singh

IN

84 Consider the following about the Law Commission of India (LCI).


1. It is a non-statutory and non-constitutional body.
2. It is headed by the incumbent Union Law Secretary.
3. It is constituted by the Parliament from time to time.
Select the correct answer using the codes below.
A. 1 only
B. 2 and 3 only
C. 1 and 3 only
D. 1, 2 and 3 only
User Answer :
Correct Answer : A
Answer Justification :
Justification:Statement 2: It is usually headed by a retired Supreme Court judge or
former Chief Justice of a high court.

(C) Insights Active Learning. | All rights reserved.

www.insightsias.com

55

TEST - 15

Statement 3: It is constituted by the Union Government from time to time. The first
commission was constituted in 1955 and since then various commissions were reconstituted every three years.
Q Source:Frequently in news

TS

IA
S

85 Consider the following statements.


1. Assertion (A): In India, only the Central government can make developmental plans.
2. Reason (R): 'Economic and social planning' falls under the Union List in the
Seventh schedule of the Constitution.
In the context of the above, which of these is correct?
A. A is correct, and R is an appropriate explanation of A.
B. A is correct, but R is not an appropriate explanation of A.
C. A is incorrect, but R is correct.
D. Both A and R are incorrect.
User Answer :
Correct Answer : D
Answer Justification :

Learning:'Economic and social planning' is a concurrent subject. Also, while


framing the 'Union', 'State' and 'Concurrent' list, allocating subjects and other
provisions, the Constitution vests power in the Union to ensure co-ordinated
development in essential fields of activity while preserving the initiative and
authority of the states in the spheres allotted to them.

SI

Q Source:MAJOR OBJECTIVES OF PLANNING: Chapter 5: Indian Economy Ramesh Singh

IN

86 The applications of Indian Regional Navigational Satellite System (IRNSS) include


1. Disaster Management
2. Vehicle tracking and fleet management
3. Mapping
Select the correct answer using the codes below.
A. 1 and 2 only
B. 2 and 3 only
C. 2 only
D. 1, 2 and 3
User Answer :
Correct Answer : D
Answer Justification :

(C) Insights Active Learning. | All rights reserved.

www.insightsias.com

56

TEST - 15

Learning:This satellite system aims to provide real-time data on the position of


objects to aid road, air and maritime traffic. It will also provide mapping and
tracking services.
IRNSS System consists of constellation of seven satellites of which three are
geostationary and four are non-geostationary.

Q Source:Frequently in news

IA
S

It will also give facility for terrestrial navigation aid for hikers and travellers and
visual and voice navigation for drivers.

TS

87 A constitutional amendment bill that amends major federal features can be passed by
A. Special majority in each house of the Parliament
B. Absolute Majority in any one house of the Parliament
C. Simple majority in both house of Parliament as well as in majority of State
Legislatures
D. None of the above is correct.
User Answer :
Correct Answer : D
Answer Justification :

SI

Learning:Ordinarily, the bill must be passed in each House by a special majority,


that is, a majority (that is, more than 50 per cent) of the total membership of the
House and a majority of two-thirds of the members of the House present and voting.

IN

Each House must pass the bill separately. In case of a disagreement between the two
Houses, there is no provision for holding a joint sitting of the two Houses for the
purpose of deliberation and passage of the bill.
If the bill seeks to amend the federal provisions of the Constitution, it must also be
ratified by the legislatures of half of the states by a simple majority, that is, a
majority of the members of the House present and voting. So, (d) is the only correct
answer.
Q Source:Chapter 10: Indian Polity: M Laxmikanth

88 The Parliament has passed National Waterways Bill, 2015 to clear way for the
conversion of several water bodies across India into transport Inland waterways. Apart
from rivers, which of the following can be said to comprise inland waterways?

(C) Insights Active Learning. | All rights reserved.

www.insightsias.com

57

1.
2.
3.
4.

Lakes
Canals
Creeks
Backwaters
Select the correct answer using the codes below.
A. 1, 2 and 4 only
B. 1 and 2 only
C. 3 and 4 only
D. 1, 2, 3 and 4
User Answer :
Correct Answer : D
Answer Justification :

IA
S

TEST - 15

TS

Learning:The National Waterways Bill, 2015 aims at declaring 106 additional


inland waterways as National Waterways in addition to the five existing National
Waterways.

It repeals the existing five Acts that declare the existing 5 national waterways.
These existing five national waterways are now covered under the new Bill.
Thus the total number of national waterways number will go up to 111 from
existing five national waterways.
Inland waterways mainly comprise rivers, lakes, canals, creeks and
backwaters. It extends about 14,500 km across the country.
However, potential of this mode of transport has not been fully exploited so
far.

SI

Q Source:http://www.prsindia.org/billtrack/the-national-waterways-bill-2015-3776/

IN

89 Consider the following about Convention on Conservation of Migratory Species (CMS).


1. It is also known as Bonn Convention
2. It was concluded under the aegis of the United Nations Environment Programme
(UNEP).
3. India is a party to the CMS.
4. It covers only terrestrial and avian migratory species throughout their range.
Select the correct answer using the codes below.
A. 1 and 4 only
B. 2 and 3 only
C. 1, 2 and 3 only
D. 1, 2, 3 and 4
User Answer :

(C) Insights Active Learning. | All rights reserved.

www.insightsias.com

58

TEST - 15

Correct Answer : C
Answer Justification :
Learning:The CMS is the only global and UN-based intergovernmental
organization established exclusively for the conservation and management of
terrestrial, aquatic and avian migratory species throughout their range.

IA
S

It is an intergovernmental treaty, concluded under the aegis of the United Nations


Environment Programme, concerned with the conservation of wildlife and habitats
on a global scale.
Since the Convention's entry into force, its membership has grown steadily to
include over 100 Parties from Africa, Central and South America, Asia, Europe and
Oceania.

TS

You may check the species coverage here

https://en.wikipedia.org/wiki/Convention_on_the_Conservation_of_Migratory_Spec
ies_of_Wild_Animals#Species_Coverage

Q Source:India has recently signed Raptor MoU, a Memorandum of


Understanding (MoU) on conservation of birds of prey in Africa and Eurasia.

IN

SI

90 Consider the following about the inter-state council.


1. Chief Ministers of all the states are its members.
2. Five Ministers nominated by the Prime Minister are permanent invitees to the
Council.
3. The council is a recommendatory body
4. The Council meets once biannually.
Select the correct answer using the codes below.
A. 1, 2 and 3 only
B. 2 and 4 only
C. 1, 3 and 4 only
D. 1, 2, 3 and 4 only
User Answer :
Correct Answer : A
Answer Justification :
Learning:The council is a recommendatory body on issues relating to inter-state,
Centre-state and Centre-union territories relations.

(C) Insights Active Learning. | All rights reserved.

www.insightsias.com

59

TEST - 15

It aims at promoting coordination between them by examining, discussing and


deliberating on such issues. Its duties, in detail, are as follows:

IA
S

Investigating and discussing such subjects in which the states or the centre
have a common interest;
making recommendations upon any such subject for the better coordination
of policy and action on it; and
Deliberating upon such other matters of general interest to the states as may
be referred to it by the chairman.
The Council may meet at least thrice in a year. Its meetings are held in camera and
all questions are decided by consensus.

TS

Q Source:Chapter 15: Indian Polity: M Laxmikanth

91 The highest weightage in Consumer Price Index (Combined) is constituted by


A. Fuel and light
B. Clothing and associated items
C. Food and associated items
D. Housing
User Answer :
Correct Answer : C
Answer Justification :

SI

Learning: CPI is disaggregated at the rural and urban levels. The new overall all
India CPI is a weighted average of the two.

IN

Picture below: Left column CPI Rural, Middle Column CPI Urban, and Right
column CPI Combined weights of various categories.

(C) Insights Active Learning. | All rights reserved.

www.insightsias.com

60

TEST - 15

You can read more about it here


http://www.onemint.com/2013/02/25/what-is-indias-cpi/
Q Source:Chapter 7: Indian Economy - Ramesh Singh

TS

IA
S

92 The Union Ministry of Environment, Forest and Climate Change (MoEFCC) has
released a new Four-colour Classification Scheme for industries based on their pollution
potential. Which of these industries come under Red Category?
1. Paints
2. Automobile Servicing
3. Sugar industry
4. Thermal Power Plants
Select the correct answer using the codes below.
A. 1 and 4 only
B. 1, 3 and 4 only
C. 2 and 4 only
D. 2 and 3 only
User Answer :
Correct Answer : B
Answer Justification :

IN

SI

Learning:The four-colour classification (Red, Orange, Green, White) scheme of


industrial sectors based on the Pollution Index (PI) which is a function of the (i)
Emissions (air pollutants) (ii) Effluents (water pollutants) (iii) hazardous wastes
generated and (iv) Consumption of resources.
Red category: PI score of 60 and above. These are severe polluting industries.
Total 60 industries including sugar, thermal power plants, paints and others
are under in it.
Orange category: PI score of 41 to 59. They moderately polluting industries.
Total 83 industries like coal, washeries and automobile servicing are placed
under it.

Q Source:http://envfor.nic.in/legis/ucp/ucpsch8.html
93 The aim of Satya Sodhak Samaj was to
A. Liberate the untouchable castes from exploitation and oppression
B. Promote the knowledge of the real Self
C. Liberate widows and poor women from social exploitation

(C) Insights Active Learning. | All rights reserved.

www.insightsias.com

61

TEST - 15

D. Promote the study of Vedas and ancient Indian culture


User Answer :
Correct Answer : A
Answer Justification :

IA
S

Learning:It is a society established by Jyotirao Phule in 1873. This was started as a


group whose main aim was to liberate the social shudra and untouchable castes from
exploitation and oppression.
Through his writings and activities Mahatma Phule condemned caste hierarchy and
the privileged status of priests in it.

TS

He openly condemned the inequality in the religious books, orthodox nature of


religion, exploitation of masses by the means of it, blind and misleading rituals, and
hypocrisy in the prevalent religion.
Q Source:Chapter 6: Bipin Chandra: Indias struggle for Independence

IN

SI

94 In Producer Price Index the prices that are used for calculation excludes
1. Taxes
2. Profits
3. Labour costs
4. Land Rent
Select the correct answer using the codes below.
A. 3 and 4 only
B. 1 and 2 only
C. 2 only
D. 1, 2, 3 and 4
User Answer :
Correct Answer : B
Answer Justification :
Learning:As the producers sell at higher prices to their wholesellers, so retailers
and the price increase is translated into the higher consumer prices-thus the PPI is
useful in having an idea of the consumer prices in the future.
In PPI, only basic prices are used while taxes, trade margins and transport costs are
excluded. This index is considered a better measure of inflation as price changes at
primary and intermediate stages can be tracked before it gets built into the finished
goods stage.

(C) Insights Active Learning. | All rights reserved.

www.insightsias.com

62

TEST - 15

Q Source:Chapter 7: Indian Economy - Ramesh Singh

TS

IA
S

95 Consider the following with reference to the Member of Parliament Local Area
Development Scheme (MPLADS).
1. The projects are executed by District administration and the role of MPs is limited
only upto recommendation of works.
2. Nominated MPs are not covered under the scheme.
Which of the above is/are correct?
A. 1 only
B. 2 only
C. Both 1 and 2
D. None
User Answer :
Correct Answer : A
Answer Justification :

Justification:Statement 1: Under the scheme, each MP has the choice to suggest to


the District Collector for, works to the tune of Rs. 5 Crore per annum to be taken up
in his/her constituency.

It is the responsibility of the District Authority to sanction, execute and complete the
works recommended by Members of Parliament within the stipulated time period.

SI

Statement 2: The Nominated Members of the Lok Sabha and Rajya Sabha may
select any Districts from any State in the Country for implementation of their choice
of work under the scheme.

IN

The Rajya Sabha MPs can recommend works in one or more districts in the State
from where he/she has been elected.
Q Source:MPLADS: Chapter 5: Indian Economy - Ramesh Singh

96 There was a general resentment against the Lex Loci Act that was proposed in 1845 and
passed in 1850. It was because
A. It provided for the confiscation of all property given to charitable and
religious institutions.
B. It provided the right to inherit ancestral property to Hindu converts to
Christianity.
C. Imparting religious education in local schools was declared illegal.
D. The Act mandated reservations on religious lines in public institutions.

(C) Insights Active Learning. | All rights reserved.

www.insightsias.com

63

TEST - 15

User Answer :
Correct Answer : B
Answer Justification :
Learning:The debate for a uniform civil code dates back to the colonial period in
India.

IA
S

The Lex Loci Report of October 1840 emphasised the importance and necessity of
uniformity in codification of Indian law, relating to crimes, evidences and contract
but it recommended that personal laws of Hindus and Muslims should be kept
outside such codification.
Q Source:Page 65: Chapter 6: Bipin Chandra: Indias struggle for Independence

SI

TS

97 Who among the following are in the Governing Council of the NITI Aayog?
1. Union Planning Secretary
2. Leader of Opposition of both houses
3. Governor, RBI
4. Chief Minister of all States of India
5. Minister of Home Affairs
Select the correct answer using the codes below.
A. 1, 3 and 4 only
B. 2, 4 and 5 only
C. 4 and 5 only
D. 1, 2 and 3 only
User Answer :
Correct Answer : C
Answer Justification :

IN

Learning:Chairperson: Prime Minister of India; Ex officio Members, NITI Aayog;


Vice-Chairperson, NITI Aayog; Full-time Members, NITI Aayog; Chief Ministers
of all states and UTs; Administrator of all UTs are the members of the Governing
Council.
Ex-officio Members are Minister of Home Affairs; Minister of Finance; Minister of
Corporate Affairs; Minister of Information and Broadcasting; Minister of Railways;
and Minister of Agriculture.
Full-Time Members are Shri Bibek Debroy; Shri V.K. Saraswat; and Prof. Ramesh
Chand.

(C) Insights Active Learning. | All rights reserved.

www.insightsias.com

64

TEST - 15

Q Source:Replaces Planning Commission: Chapter 5: Indian Economy - Ramesh


Singh

TS

IA
S

98 Which of the following argument(s) were made by Dadabhai Naoroji about the
economic aspects of the British rule in India?
1. The drain of wealth from India was the basic cause of Indias poverty.
2. Foreign capital should not be preferred for Indias growth.
Which of the above is/are correct?
A. 1 only
B. 2 only
C. Both 1 and 2
D. None
User Answer :
Correct Answer : C
Answer Justification :

Learning:He argued that it was the economic laws and policies of the British that
were draining India and causing crisis in every sphere especially in loss of
livelihoods and loss of precious capital that could have been used for Indias
development.

He and his contemporaries argued that using foreign capital for a nations growth
was undesirable and should be stopped. Building domestic capacity and funding it
by domestic sources was the most desirable since no drain would be involved

SI

Q Source:Page 73: Chapter 7: Bipin Chandra: Indias struggle for Independence

IN

99 Hyperinflation can have which of the following consequence(s)?


1. Loss of confidence in the domestic currency
2. Increase in demand of assets other than money
Which of the above is/are correct?
A. 1 only
B. 2 only
C. Both 1 and 2
D. None
User Answer :
Correct Answer : C
Answer Justification :
Justification:Statement 1 and 2: Inflation erodes the value of currency. For e.g. if

(C) Insights Active Learning. | All rights reserved.

www.insightsias.com

65

TEST - 15

inflation in India is 5000%, then the very next year the value of Rs. 5000 in year
2015 will be near Rs. 100 in year 2016. It also leads to depreciation in the exchange
rate of the currency.

IA
S

As a result, people lose confidence in the currency and holding it becomes a risky
proposition. So, people switch to other forms of wealth like Gold, Foreign Currency
(also known as inflation proof assets) etc.
It is one of the important reasons for the recent Gold rush in India. Introducing
inflation-indexed bonds, Gold bonds is one way of reducing the demand of
unproductive physical assets.
Q Source: Chapter 7: Indian Economy - Ramesh Singh

IN

SI

TS

100 Article 355 of the Constitution of India is an important instrument that enables the
Centre to exercise control over State administration. It imposes which of the following
duties on the Centre?
1. To protect every state against external aggression and internal disturbance
2. To protect the financial integrity of the states
3. To ensure that the government of every state is carried on in accordance with the
provisions of the Constitution
4. To resolve conflicts between States through negotiations or binding executive orders
Select the correct answer using the codes below.
A. 1 and 3 only
B. 2, 3 and 4 only
C. 4 only
D. 1, 3 and 4 only
User Answer :
Correct Answer : A
Answer Justification :
Learning:There is an old The Hindu Article that should be read for understanding
the Article 355 better.
http://www.thehindu.com/2002/05/04/stories/2002050401351200.htm
Financial emergency under article 360 of the constitution deals with the 2nd
statement.
Q Source:Chapter 14: Indian Polity: M Laxmikanth

(C) Insights Active Learning. | All rights reserved.

www.insightsias.com

66

You might also like